The expectation of absolute value of the sum of n i.i.d. random variables












1












$begingroup$


Let $varphi_i$ be a Gaussian random variable such that
$$varphi_i sim N(0,sigma^2), quad i = 1,2,ldots,n.$$
What's the expectation:
$$Eleft(left | sum_{i=1}^n e^{j varphi_i} right |right) $$
where $left | cdot right |$ is the absolute value operation and $j = sqrt{-1}$.










share|cite|improve this question









$endgroup$












  • $begingroup$
    I don't know if there is a nice closed form for the absolute value but for the square of the absolute value the expectation is $frac{n (n-1)}{exp left(sigma ^2right)}+n$.
    $endgroup$
    – JimB
    Jan 1 at 19:55
















1












$begingroup$


Let $varphi_i$ be a Gaussian random variable such that
$$varphi_i sim N(0,sigma^2), quad i = 1,2,ldots,n.$$
What's the expectation:
$$Eleft(left | sum_{i=1}^n e^{j varphi_i} right |right) $$
where $left | cdot right |$ is the absolute value operation and $j = sqrt{-1}$.










share|cite|improve this question









$endgroup$












  • $begingroup$
    I don't know if there is a nice closed form for the absolute value but for the square of the absolute value the expectation is $frac{n (n-1)}{exp left(sigma ^2right)}+n$.
    $endgroup$
    – JimB
    Jan 1 at 19:55














1












1








1


0



$begingroup$


Let $varphi_i$ be a Gaussian random variable such that
$$varphi_i sim N(0,sigma^2), quad i = 1,2,ldots,n.$$
What's the expectation:
$$Eleft(left | sum_{i=1}^n e^{j varphi_i} right |right) $$
where $left | cdot right |$ is the absolute value operation and $j = sqrt{-1}$.










share|cite|improve this question









$endgroup$




Let $varphi_i$ be a Gaussian random variable such that
$$varphi_i sim N(0,sigma^2), quad i = 1,2,ldots,n.$$
What's the expectation:
$$Eleft(left | sum_{i=1}^n e^{j varphi_i} right |right) $$
where $left | cdot right |$ is the absolute value operation and $j = sqrt{-1}$.







probability statistics absolute-value expected-value






share|cite|improve this question













share|cite|improve this question











share|cite|improve this question




share|cite|improve this question










asked Jan 1 at 15:32









Gertsen YuanGertsen Yuan

84




84












  • $begingroup$
    I don't know if there is a nice closed form for the absolute value but for the square of the absolute value the expectation is $frac{n (n-1)}{exp left(sigma ^2right)}+n$.
    $endgroup$
    – JimB
    Jan 1 at 19:55


















  • $begingroup$
    I don't know if there is a nice closed form for the absolute value but for the square of the absolute value the expectation is $frac{n (n-1)}{exp left(sigma ^2right)}+n$.
    $endgroup$
    – JimB
    Jan 1 at 19:55
















$begingroup$
I don't know if there is a nice closed form for the absolute value but for the square of the absolute value the expectation is $frac{n (n-1)}{exp left(sigma ^2right)}+n$.
$endgroup$
– JimB
Jan 1 at 19:55




$begingroup$
I don't know if there is a nice closed form for the absolute value but for the square of the absolute value the expectation is $frac{n (n-1)}{exp left(sigma ^2right)}+n$.
$endgroup$
– JimB
Jan 1 at 19:55










1 Answer
1






active

oldest

votes


















0












$begingroup$

The expectation you are interested in is equivalent to



$$Eleft[sqrt{n+2sum_{j=1}^{n-1}sum_{k=j+1}^n (cos varphi_j cos varphi_k+sinvarphi_j sinvarphi_k)}right]$$



I don't believe there is a nice closed-form solution for that expectation. So I think you'll need to perform simulations or numerically integrate to get solutions.



If you were interested in



$$Eleft[n+2sum_{j=1}^{n-1}sum_{k=j+1}^n (cos varphi_j cos varphi_k+sinvarphi_j sinvarphi_k)right]$$



then there is a closed-form solution:



$$n+n(n-1)e^{-sigma^2}$$






share|cite|improve this answer









$endgroup$













    Your Answer





    StackExchange.ifUsing("editor", function () {
    return StackExchange.using("mathjaxEditing", function () {
    StackExchange.MarkdownEditor.creationCallbacks.add(function (editor, postfix) {
    StackExchange.mathjaxEditing.prepareWmdForMathJax(editor, postfix, [["$", "$"], ["\\(","\\)"]]);
    });
    });
    }, "mathjax-editing");

    StackExchange.ready(function() {
    var channelOptions = {
    tags: "".split(" "),
    id: "69"
    };
    initTagRenderer("".split(" "), "".split(" "), channelOptions);

    StackExchange.using("externalEditor", function() {
    // Have to fire editor after snippets, if snippets enabled
    if (StackExchange.settings.snippets.snippetsEnabled) {
    StackExchange.using("snippets", function() {
    createEditor();
    });
    }
    else {
    createEditor();
    }
    });

    function createEditor() {
    StackExchange.prepareEditor({
    heartbeatType: 'answer',
    autoActivateHeartbeat: false,
    convertImagesToLinks: true,
    noModals: true,
    showLowRepImageUploadWarning: true,
    reputationToPostImages: 10,
    bindNavPrevention: true,
    postfix: "",
    imageUploader: {
    brandingHtml: "Powered by u003ca class="icon-imgur-white" href="https://imgur.com/"u003eu003c/au003e",
    contentPolicyHtml: "User contributions licensed under u003ca href="https://creativecommons.org/licenses/by-sa/3.0/"u003ecc by-sa 3.0 with attribution requiredu003c/au003e u003ca href="https://stackoverflow.com/legal/content-policy"u003e(content policy)u003c/au003e",
    allowUrls: true
    },
    noCode: true, onDemand: true,
    discardSelector: ".discard-answer"
    ,immediatelyShowMarkdownHelp:true
    });


    }
    });














    draft saved

    draft discarded


















    StackExchange.ready(
    function () {
    StackExchange.openid.initPostLogin('.new-post-login', 'https%3a%2f%2fmath.stackexchange.com%2fquestions%2f3058587%2fthe-expectation-of-absolute-value-of-the-sum-of-n-i-i-d-random-variables%23new-answer', 'question_page');
    }
    );

    Post as a guest















    Required, but never shown

























    1 Answer
    1






    active

    oldest

    votes








    1 Answer
    1






    active

    oldest

    votes









    active

    oldest

    votes






    active

    oldest

    votes









    0












    $begingroup$

    The expectation you are interested in is equivalent to



    $$Eleft[sqrt{n+2sum_{j=1}^{n-1}sum_{k=j+1}^n (cos varphi_j cos varphi_k+sinvarphi_j sinvarphi_k)}right]$$



    I don't believe there is a nice closed-form solution for that expectation. So I think you'll need to perform simulations or numerically integrate to get solutions.



    If you were interested in



    $$Eleft[n+2sum_{j=1}^{n-1}sum_{k=j+1}^n (cos varphi_j cos varphi_k+sinvarphi_j sinvarphi_k)right]$$



    then there is a closed-form solution:



    $$n+n(n-1)e^{-sigma^2}$$






    share|cite|improve this answer









    $endgroup$


















      0












      $begingroup$

      The expectation you are interested in is equivalent to



      $$Eleft[sqrt{n+2sum_{j=1}^{n-1}sum_{k=j+1}^n (cos varphi_j cos varphi_k+sinvarphi_j sinvarphi_k)}right]$$



      I don't believe there is a nice closed-form solution for that expectation. So I think you'll need to perform simulations or numerically integrate to get solutions.



      If you were interested in



      $$Eleft[n+2sum_{j=1}^{n-1}sum_{k=j+1}^n (cos varphi_j cos varphi_k+sinvarphi_j sinvarphi_k)right]$$



      then there is a closed-form solution:



      $$n+n(n-1)e^{-sigma^2}$$






      share|cite|improve this answer









      $endgroup$
















        0












        0








        0





        $begingroup$

        The expectation you are interested in is equivalent to



        $$Eleft[sqrt{n+2sum_{j=1}^{n-1}sum_{k=j+1}^n (cos varphi_j cos varphi_k+sinvarphi_j sinvarphi_k)}right]$$



        I don't believe there is a nice closed-form solution for that expectation. So I think you'll need to perform simulations or numerically integrate to get solutions.



        If you were interested in



        $$Eleft[n+2sum_{j=1}^{n-1}sum_{k=j+1}^n (cos varphi_j cos varphi_k+sinvarphi_j sinvarphi_k)right]$$



        then there is a closed-form solution:



        $$n+n(n-1)e^{-sigma^2}$$






        share|cite|improve this answer









        $endgroup$



        The expectation you are interested in is equivalent to



        $$Eleft[sqrt{n+2sum_{j=1}^{n-1}sum_{k=j+1}^n (cos varphi_j cos varphi_k+sinvarphi_j sinvarphi_k)}right]$$



        I don't believe there is a nice closed-form solution for that expectation. So I think you'll need to perform simulations or numerically integrate to get solutions.



        If you were interested in



        $$Eleft[n+2sum_{j=1}^{n-1}sum_{k=j+1}^n (cos varphi_j cos varphi_k+sinvarphi_j sinvarphi_k)right]$$



        then there is a closed-form solution:



        $$n+n(n-1)e^{-sigma^2}$$







        share|cite|improve this answer












        share|cite|improve this answer



        share|cite|improve this answer










        answered Jan 2 at 0:46









        JimBJimB

        55537




        55537






























            draft saved

            draft discarded




















































            Thanks for contributing an answer to Mathematics Stack Exchange!


            • Please be sure to answer the question. Provide details and share your research!

            But avoid



            • Asking for help, clarification, or responding to other answers.

            • Making statements based on opinion; back them up with references or personal experience.


            Use MathJax to format equations. MathJax reference.


            To learn more, see our tips on writing great answers.




            draft saved


            draft discarded














            StackExchange.ready(
            function () {
            StackExchange.openid.initPostLogin('.new-post-login', 'https%3a%2f%2fmath.stackexchange.com%2fquestions%2f3058587%2fthe-expectation-of-absolute-value-of-the-sum-of-n-i-i-d-random-variables%23new-answer', 'question_page');
            }
            );

            Post as a guest















            Required, but never shown





















































            Required, but never shown














            Required, but never shown












            Required, but never shown







            Required, but never shown

































            Required, but never shown














            Required, but never shown












            Required, but never shown







            Required, but never shown







            Popular posts from this blog

            Quarter-circle Tiles

            build a pushdown automaton that recognizes the reverse language of a given pushdown automaton?

            Mont Emei